Efectos de un estado de vacío no invariante de Lorentz

Estoy aquí preguntando sobre experimentos reales o de pensamiento (es decir, efectos físicos) donde, al menos en principio, uno puede ver alguna consecuencia de un estado de vacío no invariante de Lorentz en una teoría invariante de Poincaré.

Déjame desarrollar la pregunta. Supongamos una teoría en la que el hamiltoniano (que cierra el álgebra de Poincaré con el resto de generadores) H actúa de manera no trivial en el estado de vacío | 0 > , donde 'no trivialmente' simplemente significa:

H | 0 >= mi | 0 >

con mi una constante positiva. Dado que el hamiltoniano se transforma como el componente temporal de un vector de 4, el estado de vacío no es invariante de Lorentz. Por lo tanto, la teoría no es invariante de Lorentz (generalmente se afirma que esta es una condición adicional además del álgebra de Poincaré). Sin embargo, no soy capaz de ver ninguna consecuencia de este hecho. Creo que esto no afecta ninguna sección transversal o tasa de descomposición.

Creo que se puede redefinir el hamiltoniano de modo que H = H mi y H | 0 >= 0 (esto no afecta el álgebra de Poincaré si también se redefinen correctamente los generadores de impulso). Sé que esto parece obvio (esta redefinición generalmente se realiza en cuantización canónica cuando uno no adopta el orden normal), pero acabo de leer esto en este foro:

https://física.stackexchange.com/a/8360/10522

Sin embargo, en la relatividad especial, la energía es el componente de tiempo de un vector de 4 y es muy importante si es cero o distinto de cero. En particular, la energía del espacio vacío de Minkowski tiene que ser exactamente cero porque si fuera distinto de cero, el estado no sería invariante de Lorentz: las transformaciones de Lorentz transformarían la energía distinta de cero (componente de tiempo de un vector) en un impulso distinto de cero ( componentes espaciales).

Uno tiene una familia de estados de vacío relacionados por transformaciones de Poincaré que son unitarios, no creo que esto sea un problema... ¿ustedes qué opinan?

Agregado: 1) No estoy pensando en un Lagrangiano invariante de Poincaré con un potencial de la forma ( A 2 ( X ) v ) 2 , dónde A m ( X ) es un campo vectorial que adquiere un valor esperado de vacío v . Suponga que cada campo tiene cero vev.

2) Estoy buscando efectos de violación de Lorentz en lugar de efectos de energía de vacío a menos que argumente que estos efectos de energía de vacío (cambio de cordero, emisión espontánea, etc.) rompen la simetría de Lorentz.

Creo que no es exactamente la misma pregunta. El hamiltoniano de mi teoría es el relativista especial con una constante añadida. Voy a editar la pregunta para que quede más claro.
Si redefines el hamiltoniano y el impulso, no necesitas meterte con el Boost. En QFT tienes mi 0 = ω k 2 y PAG 0 = k 2 ...así que individualmente, cada redefinición de momento coincide con la redefinición de energía ;)
Gracias, @Nogueira. No puedo entender lo que quieres decir, ¿puedes explicar tu punto?
@Drake Si toma el orden normal del hamiltoniano, todavía tiene un generador de traducción y las relaciones de conmutación no cambian.

Respuestas (2)

La formulación de esta pregunta supone que es imposible tener un estado de vacío donde 0 | H | 0 > 0 sin violar la invariancia de Lorentz. Esto no es verdad. Generalmente, cuando hay densidad de energía en el vacío, se tiene la presión adecuada para mantener la invariancia de Lorentz, porque el tensor de tensión termina siendo proporcional a gramo m v es invariante de Lorentz.

En el espacio infinito, la energía sería infinita, ya que es una densidad de energía finita. Si cortas la teoría en una caja grande para regular la energía, obtienes un Lorentz rompiendo la energía total en el vacío, pero la ruptura de la invariancia de Lorentz solo proviene del hecho de que hay paredes o identificaciones que seleccionan un Lorentz. marco. Si aumenta la caja, tiene un impulso en la caja, pero eso es solo porque las presiones en los bordes de la caja aumentada ya no están equilibradas, por lo que hay un impulso neto proveniente de las paredes de la caja, o si es periódico, tiene un impulso neto de los límites periódicos móviles.

Entonces, no es cierto que la energía del vacío rompa la invariancia de Lorentz. Esto es un poco contrario a la intuición, porque estamos acostumbrados a que las energías se localicen en una partícula, por lo que la presión no puede moverse constantemente. En el vacío, la energía está en todas partes, y puede haber tensiones de presión que mantienen invariable toda la formulación de Lorentz.

Esta es la razón por la que se dice que el infinito de la energía del vacío en las teorías de campo conduce a la renormalización de la constante cosmológica, no a la ruptura de Lorentz.

Buena respuesta y buen punto el de la proporcionalidad entre el tensor de energía-momentum y la métrica de Minkowski (voto a favor para usted), sin embargo, no estoy seguro de que su respuesta sea correcta (tal vez me estoy perdiendo algo). Sé que una caja rompería Lorentz, así que supongamos que el volumen es infinito y el tensor de impulso de energía es proporcional a la métrica de Minkowski y sus componentes son finitos.
Continuación: Para tener una teoría invariante de Lorentz, se necesita un estado de vacío invariante, por lo que B i | 0 >= 0 , dónde B i es el generador de impulso. Y así uno debería tener < 0 | H | 0 >= 0 ya que el álgebra de Poincaré contiene [ B i , PAG j ] H d i j . En otras palabras, si el estado de vacío lleva el vector energía-momento ( mi , 0 ) (con mi diferente de cero), el vacío no puede ser invariante de Lorentz. Este argumento sugiere que la energía del vacío (¿renormalizada?) debe ser cero en una teoría invariante de Lorentz.
Un punto más: el hecho de que el tensor de energía-momentum (=tensor de estrés) sea proporcional a la métrica de Minkowski es una consecuencia de tener un funcional de acción invariante de Poincaré, pero no dice nada sobre las simetrías del estado de vacío, ¿verdad? ?
@drake: Uno también podría tener 0 | H 0 = ± consistentemente --- esa es la densidad de energía constante y una constante cosmológica. Por supuesto, no puedes tener una energía finita --- el espacio es infinito, tienes una densidad finita. Tienes razón en que si es un valor finito, entonces debe ser cero, pero no es profundo en absoluto.
Estimado Ron, B i = B i , PAG i | 0 > finito, B i | 0 >= 0 , [ B i , PAG j ] = i H d i j < 0 | H | 0 >= 0 . Simplemente tome el valor esperado de vacío del conmutador.
@drake: entiendo el argumento formal, no funciona cuando tienes valores propios infinitos. Es la misma falacia que en el famoso argumento de Schwinger acerca de que la conservación de la carga no funciona. En un cuadro, solo obtienes que el vacío no es invariante. Es perfectamente consistente tener energía infinita en el vacío, y es invariante de Lorentz, porque tienes una presión igual a la densidad de energía. Sus conmutadores simplemente están reafirmando las propiedades de transformación de Lorentz del tensor de energía de tensión y sus integrales.
En la teoría física (una vez regularizada y renormalizada), en el espacio-tiempo de Minkowski (sin límites), el vacío o energía de punto cero debe ser cero para preservar la invariancia de Lorentz. ¿Aceptar?
@drake: ¡No! ¿Por qué tengo que repetirlo? Su argumento es defectuoso . esta mal _ La energía del punto cero es cero, más infinito con una densidad de energía positiva finita, o menos infinito con una energía negativa finita. Todos estos son consistentes con la invariancia de Lorentz, sus juegos formales no funcionan porque el operador de energía lo saca del espacio formal de Hilbert donde sus operaciones están justificadas (actuar con H en el vacío genera infinitas partículas). En este caso, quizás sea útil la pedantería rigurosa que la gente introduce en AQFT.
@drake: cometí el mismo error hace casi exactamente 20 años, y alguien en Internet me corrigió y explicó esto. Es un error común argumentar que H=0 de la invariancia de Lorentz. Lo cierto es que el tensor de tensión local debe ser proporcional a gramo m v .

En la teoría axiomática cuántica de campos, se supone que existe un único estado invariante (proyectivo) de Poincaré. Un representante normalizado arbitrario | 0 de este estado se llama el estado de vacío. La invariancia de Poincaré implica que para todos X , el estado mi X PAG / | 0 es múltiplo de | 0 . Esto implica que PAG | 0 = pag | 0 para algunos pag . redefiniendo PAG como PAG pag da otra representación del grupo de Poincaré en el que el estado de vacío tiene cero 4 impulsos, como se suele suponer.

Por otro lado, Rn Maimón considera una situación no cubierta por la QFT axiomática, donde un estado | 0 él llama (en mi opinión erróneamente) que el estado de vacío no está en el dominio del grupo de traducción. De este modo mi X PAG / | 0 es (desde distinto de cero X ) no es un vector de estado en el espacio físico de Hilbert (sino una distribución adecuada), el valor esperado de vacío del vector de impulso no existe y los argumentos de covarianza habituales se rompen. Tal como lo entiendo, él considera que esta es la situación en un universo infinito con una densidad de momento local finita pero una energía total infinita. Sin embargo, en tal situación, la invariancia de Poincaré ya no es relevante. De hecho, en la gravedad cuántica, no parece haber una noción independiente del observador de un estado de vacío, debido al efecto Unruh. En cambio, opne tiene una familia de estados de Hadamard que, en conjunto, reemplazan el estado de vacío del QFT plano. Esta familia en su conjunto es de hecho invariante de Poincaré, incluso invariante bajo el grupo de difeomorfismos que conservan el volumen.